• C++ Programming for Financial Engineering
    Highly recommended by thousands of MFE students. Covers essential C++ topics with applications to financial engineering. Learn more Join!
    Python for Finance with Intro to Data Science
    Gain practical understanding of Python to read, understand, and write professional Python code for your first day on the job. Learn more Join!
    An Intuition-Based Options Primer for FE
    Ideal for entry level positions interviews and graduate studies, specializing in options trading arbitrage and options valuation models. Learn more Join!

A small question

Joined
8/24/12
Messages
7
Points
11
Hi

I start self-learning statistics and probability and came across a small question. Could somebody help?

Let A, B, C be three events. Let B denotes the complement of B.

How can I represent P(B| A ^ C) in terms of P(A| B ^ C)
 
i'm assuming ^=intersect
\(P(B|A^C)=1-P(B|A^C)=1-P(A^B^C)/P(A^C)\)
\(=1-[P(A^B^C)/P(A^C)]*(P(B^C)/P(B^C))=1-P(A| B ^ C)*P(B^C)/P(A^C)\)
is that what you want?
 
Oops. Yes, ^ = interest. But the result is not exactly what I want.
The whole question is : Let A, B, C be three events such that P(A ^ C) and P(B ^ C) are both strictly between 0 and 1 and let B denotes the complement of B. Prove that: P(B|A^C) = P(A|B^C) * P(B|C) / P(A|C). I managed to do this proof.

The next question is to express P(A|C) in terms of P(A|B^C), P(A|B^C) and P(B|C)

Here I used the result from the prove question. So
P(A|C) = P(A|B^C) * P(B|C) / P(B|A^C). Comparing to the conditions given, the terms P(B|A^C) and P(A|B^C) are different. That is why I asked the simplified question. Your solution introduced the new terms P(A^C) and P(B^C)?

Anyway, thanks for your reply..!!! Further explanation will be appreciated.
 
Here you go
\( P(A|C) = \frac{ P(A\cap C) }{ P(C) }=\frac{ P(A\cap C) }{ P(C) }\frac{ P(B\cap C) }{ P(B\cap C) } \)
\(=[ P(A \cap B \cap C) + P( A \cap \bar{B} \cap C )]\frac{ P(B|C) }{ P(B \cap C)}\)
\(= P(A| B \cap C)P(B|C)+ P(A | \bar{B} \cap C )P(B|C)\)
 
Here you go
\( P(A|C) = \frac{ P(A\cap C) }{ P(C) }=\frac{ P(A\cap C) }{ P(C) }\frac{ P(B\cap C) }{ P(B\cap C) } \)
\(=[ P(A \cap B \cap C) + P( A \cap \bar{B} \cap C )]\frac{ P(B|C) }{ P(B \cap C)}\)
\(= P(A| B \cap C)P(B|C)+ P(A | \bar{B} \cap C )P(B|C)\)

Emm....I think something is wrong with the result. Where does \(P(A | \bar{B} \cap C )P(B|C)\) come from? Did you mistreat \(\bar{B}\) as \({B}\) so that you can merge \(P( A \cap \bar{B} \cap C )\) and \(P(B \cap C)\)?
 
Hi

I start self-learning statistics and probability and came across a small question. Could somebody help?

Let A, B, C be three events. Let B denotes the complement of B.

How can I represent P(B| A ^ C) in terms of P(A| B ^ C)

give me the book name, page number and question. this will make it significantly easier to answer the question.
 
Oh it's not from the book. It's just an exercise.

The original question is here.

Probability.png
 
Back
Top